LSAT and Law School Admissions Forum

Get expert LSAT preparation and law school admissions advice from PowerScore Test Preparation.

User avatar
 Dave Killoran
PowerScore Staff
  • PowerScore Staff
  • Posts: 5852
  • Joined: Mar 25, 2011
|
#87783
Complete Question Explanation
(The complete setup for this game can be found here: lsat/viewtopic.php?f=157&t=1677)

The correct answer choice is (B)

The question stem indicates that only five total grants will be awarded. Thus, the grants-to-areas distribution is 2-1-1-1, with exactly 2 M awards and exactly 1 of each other area (T, W, and Y). The grants-to-quarters distribution is also 2-1-1-1, with exactly one of the quarters having two grants.

Answer choice (A): As mentioned above, there can only be one Y grant, and thus this answer choice cannot occur and is incorrect.

Answer choice (B): This is the correct answer choice.

Answer choice (C): As mentioned above, there can only be one W grant, and as there is already a W grant in the second quarter, this would result in 2 W grants. Thus, this answer choice cannot occur and is incorrect.

Answer choice (D): This answer choice cannot occur because the two M grants would be forced into the third and fourth quarters, a violation of the third rule.

Answer choice (E): This answer choice cannot occur because the two M grants would be forced into the third and fourth quarters, a violation of the third rule.
 Basia W
  • Posts: 108
  • Joined: Jun 19, 2014
|
#15694
Hello!

For the 2009 games section:

21b) I am not sure why this particular answer is right.

Thank you for your time!

Best,

Basia
 Emily Haney-Caron
PowerScore Staff
  • PowerScore Staff
  • Posts: 577
  • Joined: Jan 12, 2012
|
#15697
Hi Basia,

A is out, because just two grants are awarded in only 1 quarter, and we know from Rule 4 that it would have to be medical services.
C is out for the same reason; the last rule tells us a wildlife preservation grant is awarded in the second quarter, and we know we can't have two of them for this question.
D is out because we know W has to go in slot 2. If Y and T are both in slot 1, and we need to have two Ms (and neither can go in a slot with another grant, since the double is YT), we would have to put the two Ms in 3 and 4, but we know they can't be next to each other.
E is out for the same reason. If Y is in 1, T and W are in 2, then the Ms would have to be in 3 and 4, which violates the rule.

By process of elimination, then, the answer is B.
 Basia W
  • Posts: 108
  • Joined: Jun 19, 2014
|
#15699
Thank you for your explanation!
User avatar
 Dustin Batchelor
  • Posts: 4
  • Joined: Sep 01, 2021
|
#96256
The rules indicate no more than 6 grants offered, and "One or mor grants offered each quarter." I had trouble because I believed 3 grants in one quarter to be possible. Where is it stated that this is a bad assumption so that I can catch it next time?
Thank you for everything,
Dustin
User avatar
 Dustin Batchelor
  • Posts: 4
  • Joined: Sep 01, 2021
|
#96257
Dustin Batchelor wrote: Tue Jul 19, 2022 1:00 pm The rules indicate no more than 6 grants offered, and "One or mor grants offered each quarter." I had trouble because I believed 3 grants in one quarter to be possible. Where is it stated that this is a bad assumption so that I can catch it next time?
Thank you for everything,
Dustin
I see that 3 + 2=5 and filling the other quarters would make it 7 grants. Please don't waste your time on a response to my silly question! :ras:

Get the most out of your LSAT Prep Plus subscription.

Analyze and track your performance with our Testing and Analytics Package.